2001 AIME I Problems/Problem 2

Revision as of 17:28, 11 June 2008 by Azjps (talk | contribs) (tex cleanup)

Problem

A finite set $\mathcal{S}$ of distinct real numbers has the following properties: the mean of $\mathcal{S}\cup\{1\}$ is $13$ less than the mean of $\mathcal{S}$, and the mean of $\mathcal{S}\cup\{2001\}$ is $27$ more than the mean of $\mathcal{S}$. Find the mean of $\mathcal{S}$.

Solution

Let $x$ be the mean of $\mathcal{S}$. Let $a$ be the number of elements in $\mathcal{S}$. Then, the given tells us that $\frac{ax+1}{a+1}=x-13$ and $\frac{ax+2001}{a+1}=x+27$. Subtracting, we have

$\begin{align*}\frac{ax+2001}{a+1}-40=\frac{ax+1}{a+1} \Longrightarrow \frac{2000}{a+1}=40 \Longrightarrow a=49$ (Error compiling LaTeX. Unknown error_msg)

We plug that into our very first formula, and get:

$\begin{align*}\frac{49x+1}{50}&=x-13 \\

49x+1&=50x-650 \\

x&=\boxed{651}.\end{align*}$ (Error compiling LaTeX. Unknown error_msg)

See Also

2001 AIME I (ProblemsAnswer KeyResources)
Preceded by
Problem 1
Followed by
Problem 3
1 2 3 4 5 6 7 8 9 10 11 12 13 14 15
All AIME Problems and Solutions